Ensino SuperiorCoordenadas polares para resolução de integral dupla Tópico resolvido

Poste aqui problemas sobre assuntos estudados no Ensino Superior (exceto os cobrados em concursos públicos e escolas militares).
Avatar do usuário
dromagnolo
iniciante
Mensagens: 3
Registrado em: 07 Jul 2013, 14:27
Última visita: 25-12-18
Abr 2015 30 17:33

Coordenadas polares para resolução de integral dupla

Mensagem não lida por dromagnolo »

Olá pessoal, estou tentando resolver o seguinte exercício do Guidorizzi de aplicação de coordenadas polares para resolver integral dupla:

[tex3]\int\limits_{0}^{1}\int\limits_{1-\sqrt{1-x^{2}}}^{1+\sqrt{1-x^{2}}}xy dydx[/tex3]

primeiramente tentei a substituição: Y= y-1 e X = r cos [tex3]\theta[/tex3] , pois trata-se de uma circunferência centrada em 0,1, mas ai sempre caio no valor zero para a integral, e suspeitei de que meu intervalo para [tex3]\theta[/tex3] estivesse errado, então conferi a resposta e ele usa x=rcos [tex3]\theta[/tex3] e y = rsen [tex3]\theta[/tex3] , com 0 [tex3]\leq\theta\leq\left(\frac{\pi }{2}\right) e 0\leq[/tex3] r [tex3]\leq[/tex3] 2 sen [tex3]\theta[/tex3] .
Alguém sabe me explicar porque a minha substituição da errado e por que ele usa teta variando de 0 a pi/2??

Grata quem puder me ajudar!
Resposta

resposta = 2/3

Editado pela última vez por caju em 23 Jan 2018, 16:36, em um total de 2 vezes.
Razão: TeX --> TeX3
Avatar do usuário
Cardoso1979
6 - Doutor
Mensagens: 4006
Registrado em: 05 Jan 2018, 19:45
Última visita: 04-04-23
Localização: Teresina- PI
Agradeceu: 268 vezes
Agradeceram: 1109 vezes
Jan 2018 23 14:11

Re: Coordenadas polares para resolução de integral dupla

Mensagem não lida por Cardoso1979 »

Observe:

Solução

y = 1 + [tex3]\sqrt{1 - x²}[/tex3]

( y - 1 )^2 = ( [tex3]\sqrt{1 - x²}[/tex3] )^2

x² + ( y - 1 )^2 = 1( como você mencionou acima, trata-se de uma circunferência de raio 1 centrada em ( 0 ; 1 ) ), em outras palavras essa circunferência está localizada no 1° e 2° quadrantes, como a região está limitada pelas retas x = 0 e x = 1, logo a região que estamos procurando se dá no 1° quadrante , ou seja , trata-se de uma semicircunferēncia.

Por outro lado, vamos transformar a equação cartesiana x² + ( y - 1 )^2 = 1 numa equação polar, temos:

[ r.sen([tex3]\theta [/tex3] ) - 1 ]^2 = 1 - r².cos² [tex3](\theta) [/tex3]

r².sen²([tex3]\theta [/tex3] ) - 2r.sen([tex3]\theta [/tex3] ) + 1 = 1 - r².cos² [tex3](\theta) [/tex3]

r².[ sen² [tex3](\theta )[/tex3] + cos² [tex3](\theta) [/tex3] ] = 2r.sen [tex3]\theta [/tex3]

r² = 2r.sen [tex3]\theta [/tex3]

r = 2sen [tex3]\theta [/tex3]

Portanto, os limites de integração são: 0 [tex3]\leq [/tex3] r [tex3]\leq [/tex3] 2.sen [tex3]\theta [/tex3] (importante esboçar o gráfico para compreender melhor esses limites de integração ) e 0 [tex3]\leq \theta \leq \frac{π}{2}[/tex3] ( pois a região se dá no primeiro quadrante ).

Logo;

R = { ( r , [tex3]\theta [/tex3] ) / 0 [tex3]\leq [/tex3] r [tex3]\leq [/tex3] 2.sen [tex3]\theta [/tex3] ; 0 [tex3]\leq \theta \leq \frac{π}{2}[/tex3] }

Então;

[tex3]\int\limits_{D}\int\limits_{}^{}f( x , y )dA[/tex3] =
[tex3]\int\limits_{R}^{}\int\limits_{}^{}F(r.cos\theta , r.sen\theta )[/tex3] rdrd [tex3]\theta [/tex3] =

[tex3]\int\limits_{0}^{\frac{π}{2}}\int\limits_{0}^{2sen\theta }( rcos\theta).(rsen\theta )rdrd\theta [/tex3] =

[tex3]\int\limits_{0}^{\frac{π}{2}}\int\limits_{0}^{2sen\theta }r³.(cos\theta).(sen\theta )drd\theta [/tex3] =

[tex3]\int\limits_{0}^{\frac{π}{2}}\frac{2⁴.sen⁴\theta }{4}sen\theta .cos\theta [/tex3] =

4 [tex3]\int\limits_{0}^{\frac{π}{2}}sen^{5}\theta .cos\theta d\theta [/tex3] =

[tex3]\left(\frac{4}{6}\right)[/tex3] .[tex3]sen^{6}\left(\frac{π}{2}\right)[/tex3] =

[tex3]\left(\frac{4}{6}\right)[/tex3] .1 =

[tex3]\frac{4}{6} = \frac{2}{3}[/tex3]

Portanto, a resposta é [tex3]\frac{2}{3}[/tex3]



Nota:

x = r.cos [tex3]\theta [/tex3]

y = r.sen [tex3]\theta [/tex3]

x² + y² = r²

dydx = rdrd [tex3]\theta [/tex3]

Obs.

[tex3]\int\limits_{}^{}sen²\theta.cos\theta d\theta = \frac{sen³ \theta }{3}[/tex3] + C

[tex3]\int\limits_{}^{}sen³\theta.cos\theta d\theta = \frac{sen⁴ \theta }{4}[/tex3] + C


[tex3]\int\limits_{}^{}sen⁴\theta.cos\theta d\theta = \frac{sen^{5} \theta }{5}[/tex3] + C

Logo;

[tex3]\int\limits_{}^{}sen^{5}\theta .cos\theta d\theta = \frac{sen^{6} \theta }{6}[/tex3] + C

Abraços!!!

Editado pela última vez por Cardoso1979 em 24 Jan 2018, 00:38, em um total de 1 vez.
Responder
  • Tópicos Semelhantes
    Resp.
    Exibições
    Últ. msg

Voltar para “Ensino Superior”